You are on page 1of 79

General MRCS Recalls Paper 2 From Previous Exams

20A 28 year old man is in the surgical intensive care unit. He has
suffered a flail chest injury several hours earlier
and he was intubated and ventilated. Over the past few minutes he has
become increasingly hypoxic and is now
needing increased ventilation pressures. What is the most common
cause?
A. Pulmonary embolism
B. Cardiac tamponade
C. Fat embolism
D. Tension pneumothorax
E. Adult respiratory distress syndrome
Theme from April 2011 Exam
A flail chest segment may lacerate the underlying lung and create a flap
valve. A tension pneumothorax can be
created by intubation and ventilation in this situation. Sudden hypoxia
and increased ventilation pressure are
clues.

Theme: Audit
A. Standards based audit
B. Departmental review
C. Systems based audit
D. Operational audit
E. Financial audit
F. Peer review
Please select the most appropriate type of audit method for the situation
described. Each option may be used
once, more than once or not at all.
10. A surgical department wishes to determine whether it is using types
of prosthetic mesh material for
incisional hernia surgery in the most effective manner. Recently there
have been cases of non mesh
usage and loss of material as a result of the implants being "out of date".
You answered Financial audit
The correct answer is Systems based audit
Theme from April 2012 Exam
Theme from April 2013 Exam
This is primarily an issue of stock control. However, the system by
which the materials are used
within the theatre will need evaluation. Because it is the usage and stock
that are a problem, rather
than the sourcing the systems based audit will be more effective than an
operational audit.
11. A group of surgeons wish to determine whether patients are
receiving adequate deep vein thrombosis
prophylaxis following surgery.
You answered Departmental review
The correct answer is Standards based audit
This type of audit is widely undertaken in most trusts in the UK.
12. Surgeons are becoming increasing concerned about the adverse
results of Mrs X performing a new an
innovative operative procedure not widely practised elsewhere.
You answered Standards based audit
The correct answer is Peer review
In the situation where a surgeon performs an unfamiliar procedure a
peer review is often the best way
to evaluate the problem. This does not have to be externally based, but
often is.

Theme: Infections
A. Staphylococcus aureus
B. Streptococcus bovis
C. Clostridium perfringens
D. Clostridium dificile
E. Clostridium tetani
F. Klebsiella
G. Streptococcus pyogenes
H. Yersinia enterocolitica
I. None of the above
Please select the most likely pathogen to account for the scenario given.
Each option may be used once, more
than once or not at all.
34. A 72 year old man with peripheral vascular disease develops a
gangrenous toe. This becomes infected
and there is evidence of infection in the surrounding tissues. On clinical
palpation there is crepitus
present within the tissues.
Clostridium perfringens
Theme from January 2013 Exam
Clostridium perfringens is the most likely pathogen to be associated
with gangrene.
35. A 22 year old lady is breastfeeding her first child. One week post
partum she presents with a tender
indurated mass in the right breast.
Staphylococcus aureus
Staphylococcus aureus is the commonest cause of lactational mastitis.
36. A 45 year old man is recovering in hospital following a total hip
replacement. He develops a profuse
and watery diarrhoea. Several other patients have been suffering from
similar symptoms.
You answered Yersinia enterocolitica
The correct answer is Clostridium dificile
Clostridium dificile can spread rapidly on surgical wards. The use of
broad spectrum prophylactic
antibiotics during arthroplasty surgery can increase the risk.

Theme: Use of vasoactive drugs


A. Dopexamine
B. Dobutamine
C. Noradrenaline
D. Adrenaline
E. Milrinone
F. Dopamine
Please select the most appropriate inotrope for the scenario given. Each
option may be used once, more than
once or not at all.
2. An inotrope with mixed vaso dilating and vaso constricting properties.
You answered Noradrenaline
The correct answer is Dopamine
Dilating in the case of renal circulating and constricting in other areas.
Overall increases cardiac
output. Concept of renal dose dopamine is out of date.
3. An inotrope that is a phosphodiesterase inhibitor.
Milrinone
Milrinone works by increasing intracellular cAMP concentration.
4. An inotrope that would be useful in a 23 year old female with sepsis
secondary to pyelonephritis. She
has an increased cardiac output and decreased systemic vascular
resistance.
Noradrenaline
Theme from September 2011 Exam
In a setting of septic shock with normal or high cardiac output and
decreased SVR, a peripherally
acting vasoconstrictor such as noradrenaline would be the primary
choice.

A 56 year old lady presents with a pathological fracture of the proximal


femur. Which of the following primary
sites is the most likely source of her disease?
A. Thyroid
B. Breast
C. Kidney
D. Endometrium
E. None of the above
Theme from September 2011 exam
The correct answer is breast, because the question asks for the most
likely primary site. Breast cancer is the
commonest cause of lytic bone metastasis in women of this age,
especially from amongst those options given.

A 63 year old man undergoes a laparotomy and small bowel resection.


Twelve hours post operatively he
is noted to have a decreased urine output. Which of the hormones listed
below is most likely to be
responsible?
A. Cortisol
B. Atrial natriuretic hormone
C. Vasopressin
D. Insulin
E. Glucagon
Theme from January 2013 Exam
Vasopressin is released in increased quantities following most operative
procedures and will tend to
cause water retention. Excessive administration of intravenous fluids in
an attempt to force a diuresis
may cause fluid overload in post operative patients for this reason.

Theme: ASA scoring


A. ASA 1
B. ASA 2
C. ASA 3
D. ASA 4
E. ASA 5
The American society of anaesthesiologists physical status scoring
system is a popular method for
stratifying patients physical status. Please select the most appropriate
ASA grade for each of the
following scenarios. Each option may be used once, more than once or
not at all.
8. A 66 year old man is admitted following a collapse whilst waiting for
a bus. Clinical examination
confirms a ruptured abdominal aortic aneurysm. He is moribund and
hypotensive
ASA 5
Theme from 2009 Exam
Patients who are moribund and will not survive without surgery are
graded as ASA 5.
9. A 23 year old man with a 4cm lipoma on his flank is due to have this
removed as a daycase. He is
otherwise well.
ASA 1
Absence of co-morbidities and small procedure with no systemic
compromise will equate to an
ASA score of 1.
10. A 72 year old man is due to undergo an inguinal hernia repair. He
suffers from COPD and has an
exercise tolerance of 10 yards. He also has pitting oedema to the thighs.
ASA 4
Severe systemic disease of this nature is a constant threat to life.
Especially as he also has
evidence of cardiac failure.

A 72 year old man is due to undergo an oesophagectomy for malignancy.


His BMI is 17.5. What is the best feeding regime immediately following
surgery?
A. Total parenteral nutrition.
B. Feeding jejunostomy.
C. Feeding duodenostomy.
D. Liquid diet orally.
E. Soft solids orally.
Theme from April 2012 Exam
This patient has a condition causing poor absorption, loss of nutrients
and
high metabolism. Enteral feeds should be used where possible and many
surgeons will site a jejunostomy for this purpose. Oral diet is not
permitted
following a resection until the anastamosis has had time to heal.

patient with tachycardia and hypotension is to receive inotropes. Which


of the following conditions are
most likely to be treated with inotropes?
A. Hypovolaemic shock
B. Septic shock
C. Neurogenic shock
D. Cardiogenic shock
E. None of the above
Theme from April 2012 Exam
The term septic shock has a precise meaning and refers to refractory
systemic arterial hypotension in
spite of fluid resuscitation. Patients will therefore usually require
inotropes. Individuals suffering from
neurogenic shock will usually receive intravenous fluids to achieve a
mean arterial pressure of
90mmHg. If this target cannot be achieved then these patients will
receive inotropes. Hypovolaemic
shock requires fluids and the management of cardiogenic shock is
multifactorial and includes inotropes,
vasodilators and intra-aortic balloon pumps

Theme: Feeding options


A. Feeding jejunostomy
B. Percutaneous endoscopic gastrostomy
C. Total parenteral nutrition
D. Naso gastric feeding tube
E. Naso jejunal feeding tube
F. Normal oral intake
Please select the most appropriate method of delivering nutrition in
each of the following scenarios.
Each option may be used once, more than once or not at all.
4. A 28 year old man is comatose, from head injuries, on the
neurosurgical intensive care unit. He is
recovering well and should be extubated soon.
Naso gastric feeding tube
Theme from April 2012 Exam
Theme from January 2013 Exam
The feeding of head injured patients was reviewed in a 2008 Cochrane
report. They concluded that
the overall evidence base was poor. However, there was a trend for the
enteral route, with NG
feeding in the later stages following injury. This is contra indicated if
there are signs of basal skull
fractures.
5. A 56 year old man has undergone a potentially curative
oesophagectomy for carcinoma.
Feeding jejunostomy
Feeding jejunostomy is the standard of care in most centres. Naso
jejunal tubes are preferred by
some surgeons. However, if they become displaced the only alternative
then becomes TPN.
6. A 43 year old man is recovering from a laparoscopic low anterior
resection with loop ileostomy.
Normal oral intake
Early feeding in this situation is both safe and will enhance recovery.

A 27 year old man is involved in a road traffic accident. He is seen in the


emergency department
with chest pain. Clinical examination is essentially unremarkable and he
is discharged. He
subsequently is found dead at home. What is the most likely underlying
injury?
A. Tracheobronchial tree injury
B. Traumatic aortic disruption
C. Cardiac laceration
D. Diaphragmatic rupture
E. Rupture of the oesophagus
Theme from January 2013 Exam
Aortic injuries that do not die at the scene may have a contained
haematoma. Clinical signs are subtle
and the diagnosis may not be apparent on clinical examination. Without
prompt treatment the
haematoma usually bursts and the patient dies.

What is the opposite of relative risk?


A. P value
B. Odds ratio
C. Likelihood ratio
D. Absolute risk reduction
E. Number needed to treat
Theme from September 2012 Exam
A 53 year old man presents with an ulcerated mass at the
anal verge. A biopsy is taken and the histology demonstrates
as squamous cell carcinoma. Infection with which of the
viruses below is most likely to have contributed to the
development of the condition?
A. Human papillomavirus 7
B. Human immunodeficiency virus 1
C. Human immunodeficiency virus 2
D. Human papillomavirus 16
E. Human T-lymphotropic virus 1
Theme from January 2013 exam
Infection with human papilloma virus 16 is a risk factor for the
development of intra epithelial dysplasia of the anal skin with
subsequent increased risk of invasive malignancy.

A 32 year old man is involved in a motorcycle accident and sustains a


closed unstable spiral
tibial fracture. This is managed with an intramedullary nail. On return
to the ward he is noted to
have increasing pain in the limb and on examination the limb is swollen
and tender with pain on
passive stretching of the toes. The most likely diagnosis is:
A. Tibial nerve neuropraxia
B. Displaced tibial nail
C. Compartment syndrome
D. Deep vein thrombosis
E. Sciatic nerve injury
Theme from 2009 Exam
Theme from September 2012 Exam
Severe pain in a limb should raise suspicions of compartment syndrome
especially in tibial fractures
following fixation with intra medullary devices.

Theme: Parasitic infections


A. Giardia Infection
B. Cryptosporidium infection
C. Clonorchis sinensis infection
D. Ancyclostoma duodenale infection
E. Ascaris lumbricoides infection
F. Echinococcus granulosus infection
G. Enterobius vermicularis infection
Please select the most likely infective organism for the scenario given.
Each option may be used once,
more than once or not at all.
12. A 6 year old boy presents with symptoms of recurrent pruritus ani.
On examination there is
evidence of a small worm like structure protruding from the anus.
Enterobius vermicularis infection
Similar theme to September 2011 Exam
Infection with enterobius is extremely common. Pruritus is the main
symptom, as there is a lack
of tissue invasion it is rare for individuals to have any signs of systemic
sepsis.
13. A 58 year old man is reviewed in the clinic following a successful
cadaveric renal transplant the
previous year. He has been able to return to work as a swimming
instructor. Over the past week
he reports that he has been suffering from recurrent episodes of
diarrhoea. It has made him feel
lethargic and exhausted. Stool microscopy shows evidence of cysts.
Cryptosporidium infection
Cryptosporidium is associated with infection, particularly in those who
are immunocompromised.
Diarrhoea is the main disease. The cysts are typically identified on stool
microscopy.
14. A 25 year old man returns from a backpacking holiday in India. He
presents with symptoms of
coughing and also of episodic abdominal discomfort. Peri anal
examination is normal. Stool
microscopy demonstrates both worms and eggs within the faeces.
Ascaris lumbricoides infection
Infection with Ascaris lumbricoides usually occurs after individuals have
visited places like sub
Saharan Africa or the far east. Unlike ancyclostoma duodenale infection
there is usually evidence
of both worms and eggs in the stool. The absence of pruritus makes
enterobius less likely. The
presence of coughing may be due to the migration of the larva through
the lungs.

A 28 year old man is involved in a road traffic accident and sustains a


flail chest injury. On arrival in
the emergency department he is hypotensive. On examination he has an
elevated jugular venous pulse
and auscultation of the heart reveals quiet heard sounds. What is the
most likely diagnosis?
A. Pneumothorax
B. Myocardial contusion
C. Cardiac tamponade
D. Haemothorax
E. Ventricular septal defect
Theme from 2010 exam
The presence of a cardiac tamponade is suggested by Becks Triad:
Hypotension
Muffled heart sounds
Raised JVP

A 23 year old man is recovering from an appendicectomy. The


operation was complicated by the
presence of perforation. He is now recovering on the ward. However, his
urine output is falling and he
has been vomiting. Which of the following intravenous fluids should be
initially administered, pending
analysis of his urea and electrolyte levels?
A. Hartmans solution
B. Dextran 70
C. Pentastarch
D. Gelofusin
E. 5% Dextrose
Theme from January 2011 Exam
He will have sequestration of electrolyte rich fluids in the abdomen and
gut lumen. These are best
replaced by use of Hartmans solution in the first instance.

A 32 year old man presents to the acute surgical unit with acute
pancreatitis. He suddenly becomes
dyspnoeic and his saturations are 89% on air. A CXR shows bilateral
pulmonary infiltrates. His CVP
pressure is 16mmHg. What is the most likely diagnosis?
A. Pulmonary oedema
B. Pneumococcal pneumonia
C. Staphylococcal pneumonia
D. Pneumocystis carinii
E. Adult respiratory distress syndrome
Theme from January 2012 Exam
Acute pancreatitis is known to precipitate ARDS. ARDS is
characterised by bilateral pulmonary
infiltrates and hypoxaemia. Note that pulmonary oedema is excluded by
the CVP reading < 18mmHg.

A 29 year old female undergoes a sub total thyroidectomy. Five days


post operatively the wound
becomes erythematous and discharges pus. What is the most likely
causative organism?
A. Streptococcus pyogenes
B. Haemophilus influenzae
C. Pseudomonas aeruginosa
D. Staphylococcus aureus
E. Proteus mirabilis
Theme from 2011 Exam
In this setting Staphylococcus aureus Infection is the most likely cause.
In the UK between 2010 and
2011 the commonest cause of wound infection was enterobacter
infections (usually following cardiac or
colonic surgery). 23% of infections were due to Staph aureus, which fits
the scenario above. Infection
with the other organisms including strep pyogenes was much rarer.

Theme: Management of skin wounds


A. Immediate split thickness skin graft
B. Delayed split thickness skin graft
C. Primary closure
D. Delayed primary closure
E. Compression bandages
F. Myocutaneous flap
G. Random free flap
For each of the following injury scenarios please select the most
appropriate management. Each option
may be used once, more than once or not at all.
23. A 63 year old male is gardening when he trips and lands on a scythe.
He sustains a deep
laceration of his lateral thigh, it measures 3cm depth by 7cm length, it
penetrates down to the
bone, but no fracture is evident on imaging or examination. His co-
morbidities include type II
diabetes mellitus (diet controlled) and polymyalgia rheumatica (takes
regular low dose
prednisolone).
Delayed primary closure
Theme from September 2012 Exam
Wounds which are contaminated or have the potential to become so are
unsafe for immediate
primary closure. The combination of diabetes and steroids makes
wound complications more
likely. Despite his high risk a primary skin graft or flap is unlikely to be
a safer option. Either may
be used at a later date in the event that delayed primary closure is
unsuccessful.
24. A 71 year old lady trips over and falls landing on her left shin. She
sustains a large pretibial
laceration of her leg.
You answered Delayed split thickness skin graft
The correct answer is Immediate split thickness skin graft
Pretibial lacerations do not heal well. Simple apposition of skin edges
almost always fails due to
poor quality dermal tissues and underlying haematoma. Debridement a
primary grafting usually
gives the best results.
25. A 73 year old lady presents with an ulcer overlying her medial
malleolus. It is painless and has
been present for 4 months. She has oedema of the lower limbs and her
ABPI measures 0.9.
You answered Myocutaneous flap
The correct answer is Compression bandages
This is likely to be a venous leg ulcer. These are typically managed using
compression bandages.
Contra indications to this technique include peripheral vascular disease
(not present here).

Which organism is most likely to cause osteomyelitis in children with


sickle cell disease?
A. Group A Streptococcus species
B. Staphylococcus aureus
C. Enterobacter
D. Salmonella species
E. Group B Streptococcus species
Theme from January 2013 Exam
Worldwide the Salmonella species remains the most common infecting
organism in children with
osteomyelitis.
A 22 year old female is admitted to A&E with a splenic rupture. She has
not been involved in any
trauma. What infection can cause spontaneous splenic rupture?
A. Measles
B. Mumps
C. Influenza
D. Epstein-Barr virus
E. Rubella
Theme from 2009 and 2011 Exams
Epstein - Barr Virus may cause generalised lymphadenopathy. This
may be associated with
splenomegaly. This enlargement has been associated with spontaneous
rupture.

A 23 year old man sustains a severe facial fracture and reconstruction is


planned. Which of the
following investigations will facilitate pre-operative planning?
A. Mandibular tomography
B. Magnetic resonance scan of face
C. Skull X-ray
D. Computerised tomography of the head
E. Orthopantomogram
Theme from 2011 Exam
Significant facial fractures may have intracranial communication. CT
scanning will allow delineation of
injury extent and 3D reconstruction images can be created. An
Orthopantomogram (OPT) will provide
good images of mandible and surrounding bony structures but will not
give intracranial detail. A skull xray
lacks the detail for modern practice.

Theme: Management of bleeding


A. Ligate vessel
B. Underrun vessel
C. Use of diathermy
D. Application of surgicell
E. Digital pressure

In each of the following scenarios the surgeon has encountered bleeding.


Please select the most
appropriate immediate management of the situation from the list below.
Each option may be
used once, more than once or not at all.
32. A 23 year old man is undergoing an open appendicectomy. The
surgeons extend the incision
medially and suddenly encounter troublesome bleeding.
Ligate vessel
Theme from April 2012 Exam
Medial extension of an appendicectomy incision carries the risk of
injury to the inferior epigastric
artery. This can bleed briskly and is best managed by ligation.
33. A 45 year old man is undergoing a laparotomy and following
incision of the skin multiple
bleeding points are identified in the dermis and sub dermal tissues.
Use of diathermy
Multiple bleeding points are best managed through the use of diathermy.
34. A 38 year old lady is undergoing a laparotomy when the surgeons
damage the common iliac vein
whilst commencing a pelvic dissection.
You answered Underrun vessel
The correct answer is Digital pressure
Major venous bleeding such as this should be controlled with digital
pressure in the first instance.
The definitive management will usually consist of suturing the defect
closed with prolene sutures.
Transection of the common iliac vein will necessitate a major venous
reconstruction.

Theme: Paediatric emergencies


A. Manage conservatively
B. Immediate emergency theatre
C. Treat in emergency department
D. Treat in emergency department under sedation
E. Operate on next emergency list
Please select the most appropriate intervention for the scenario given.
Each option may be used once,
more than once or not at all.
54. A 3 year old child inserts a crayon into their external auditory
meatus. Attempts to remove it have
not been successful.
You answered Treat in emergency department under sedation
The correct answer is Operate on next emergency list
Theme from September 2011 Exam
They would not tolerate removal in the emergency department. The
tympanic membrane should
be carefully inspected and again this will be easier under general
anaesthesia.
55. A 2 year old accidentally inhales a peanut. They arrive in the
emergency department extremely
distressed and cyanotic. Imaging shows it to be lodged in the left main
bronchus.
Immediate emergency theatre
As they are cyanosed it requires immediate removal and this should be
undertaken in a fully
staffed theatre. Ideally a rigid bronchoscopy should be performed.
56. A 10 year old boy is shot in the head with an airgun pellet. He is
concerned that he will get into
trouble and the injury remains concealed for 10 days. Imaging using CT
scanning shows it to be
lodged in the frontal lobe.
You answered Operate on next emergency list
The correct answer is Manage conservatively
The pellet is small and no serious injury has occurred at this stage. This
should therefore be
managed conservatively.

Theme: Bleeding disorders


A. Vitamin K deficiency
B. von Willebrand's disease
C. Acquired haemophilia
D. Haemophilia B
E. Protein C deficiency
F. Disseminated intravascular coagulation
G. Factor V Leiden
H. Excess heparin
I. Warfarin overdose
What is the most likely diagnosis for the scenario given? Each option
may be used once, more than once
or not at all.
80. A 33 year old female is admitted for varicose vein surgery. She is fit
and well. After the
procedure she is persistently bleeding. She is known to have
menorrhagia. Investigations show a
prolonged bleeding time and increased APTT. She has a normal PT and
platelet count.
von Willebrand's disease
Bleeding post operatively, epistaxis and menorrhagia may indicate a
diagnosis of vWD.
Haemoarthroses are rare. The bleeding time is usually normal in
haemophilia (X-linked) and
vitamin K deficiency.
81. A 70 year old coal miner presents with 3 weeks of haematuria and
bruising. He is normally fit and
well. He is on no medications. His results reveal:
Hb 9.0
WCC 11
Pl 255
PT 16 (normal)
APTT 58 (increased)
Thrombin time 20 (normal).
Acquired haemophilia
Factor 8 acquires an inhibitor. The elderly, pregnancy, malignancy and
autoimmune conditions
are associated with acquired haemophilia. Prolonged APTT is key to the
diagnosis. Management
involves steroids.
82. A 33 year old female is attends the day unit for elective varicose vein
surgery. She has previously
had recurrent pulmonary embolic events. After the procedure she is
persistently bleeding. Her
APTT is 52 (increased).
You answered Warfarin overdose
The correct answer is Factor V Leiden
A combination of thromboembolism and bleeding in a young woman
should raise the possibility
of antiphospholipid syndrome. Other features may include foetal loss,
venous and arterial
thrombosis and thrombocytopenia. Protein C is not associated with a
prolonged APTT.
Theme from January 2012 exam

Theme: Management of skin lesions


A. 5mm punch biopsy
B. Shave biopsy
C. Excisional biopsy
D. Wide excision of 5cm
E. Tru cut biopsy
F. Incisional biopsy

For the skin lesions described please select the most appropriate
management option. Each
option may be used once, more than once or not at all.
33. An 83 year old lady presents with multiple patches of pigmented
irregular, superficial lesions over
the torso. They do not bleed but have become increasingly itchy.
Shave biopsy
Theme from April 2012 Exam
This is most likely to be seborrhoeic warts. These are usually
superficially sited and are best
managed with shave biopsy and cautery.
34. A 65 year old man presents with a 5cm ulcerated area over his
medial malleolus.
5mm punch biopsy
This is likely to be a venous ulcer and should usually be managed with
compression bandaging if
there is no arterial compromise. Long standing lesions may be
complicated by the development of
malignancy and for this reason a punch biopsy of long standing or non
healing lesions is
advisable.
35. A 23 year old lady presents with an itchy, bleeding pigmented lesion
on her right thigh.
You answered Wide excision of 5cm
The correct answer is Excisional biopsy
This may represent a malignant melanoma. Complete excision is
required to allow accurate
histological assessment. If the diagnosis is confirmed then re-excision of
margins may be
required. Clearly if the lesion is benign then no further action is
required.

A 22 year old man presents with a 5 day history of sore throat, malaise
and fatigue. On examination he
has a large peritonsillar abscess. What is the most likely underlying
infective organism?
A. Epstein Barr Virus
B. Streptococcus pyogenes
C. Cytomegalovirus
D. Moraxella catarrhalis
E. Streptococcus viridans
Similar theme in September 2011 Exam
Quinsy usually occurs as a result of bacterial tonsillitis and the most
common cause of bacterial
tonsillitis is streptococcal organisms.

A 21 year old man falls down a ravine whilst skiing and is trapped for
several hours. He is finally
brought to the emergency department profoundly hypothermic with a
core temperature of 29oC.
Which method is most effective at raising the core temperature?
A. Re-warming with electric blankets
B. Increasing the room temperature
C. Instillation of warm intravesical fluid
D. Instillation of warmed rectal fluid
E. Instillation of warmed intra peritoneal fluid

Theme from 2011 Examination


Visceral cavity re-warming be it lung or abdomen (or both) provides
rapid rewarming. Only
extracorporeal circulatory devices provide faster rates of re-warming.

A 21 year old man is undergoing an inguinal hernia repair and receives


a dose of intravenous coamoxiclav.
He is reported to have a penicillin allergy. Over the next few minutes his
vital signs are:
Pulse - 130bpm, blood pressure- 60/40mmHg. What is the first line
treatment?
A. Hydrocortisone 100mg IV
B. Adrenaline 1:1000 IV
C. Chlorpheniramine 10mg IV
D. Adrenaline 1:1000 IM
E. Adrenaline 1:10000 IV
Theme from 2009 Exam
The first line treatment of anaphylactic shock is intra muscular
adrenaline.

A 43 year old man has symptoms of carcinoid syndrome. Which of the


following is the most effective
therapeutic agent in controlling the symptoms?
A. Atenolol
B. Octreotide
C. Glucagon
D. Somatostatin
E. Spironolactone
Theme from April 2012 Exam
Octreotide is the usual treatment for carcinoid syndrome. Somatostatin
inhibits the release of a number
of gut hormones. Octreotide is the synthetic alternative to somatostatin
and thus the most appropriate
therapeutic agent.

A 22 year old man is brought to the emergency department. He was


found lying unconscious on his right
arm and it is evident that he has taken a temazepam overdose. His right
arm is mottled in colour and
swollen, his hand is insensate and stiff. What substance is most likely to
be present in the urine in
increased quantities?
A. Protein
B. Haemoglobin
C. Myoglobin
D. Erythrocytes
E. Lymphocytes
Theme from 2011 Exam
This man is likely to have muscle death secondary to compartment
syndrome. This will result in muscle
breakdown and release of myoglobin. This may accumulate in the
kidney and result in renal failure.

A 72 year old man is due to undergo an oesophagectomy for malignancy.


His BMI is 17.5. What is the
best feeding regime immediately following surgery?
A. Total parenteral nutrition.
B. Feeding jejunostomy.
C. Feeding duodenostomy.
D. Liquid diet orally.
E. Soft solids orally.
Theme from April 2012 Exam
A 48 year old man is recovering on the high dependency unit following a
long and complex
laparotomy. His preoperative medication includes an ACE inhibitor for
blood pressure control.
For the past two hours he has been oliguric with a urine output of
10ml/hr-1. What the most
appropriate course of action?
A. Stop the ACE inhibitor
B. Administer a fluid challenge
C. Start an infusion of nor adrenaline
D. Administer intravenous frusemide
E. Insert a Swann-Ganz Catheter

Theme from April 2012 Exam


Hypovolaemia is the most likely cause for oliguria and a fluid challenge
is the most appropriate
action. Blind administration of inotropes to hypovolaemic patients is
unwise, with the possible
exception of cardiac patients.

Theme: Hernias
A. Umbilical hernia
B. Para umbilical hernia
C. Morgagni hernia
D. Littres hernia
E. Bochdalek hernia
F. Richters hernia
G. Obturator hernia
Please select the hernia that most closely matches the description given.
Each option may be used once, more
than once or not at all.
28. A 1 day old infant is born with severe respiratory compromise. On
examination he has a scaphoid
abdomen and an absent apex beat.
Bochdalek hernia
Theme from 2011 exam
The large hernia may displace the heart although true dextrocardia is
not present. The associated
pulmonary hypoplasia will compromise lung development.
29. A 2 month old infant is troubled by recurrent colicky abdominal
pain and intermittent intestinal
obstruction. On imaging the transverse colon is herniated into the
thoracic cavity, through a mid line
defect.
Morgagni hernia
Morgagni hernia may contain the transverse colon. Unless there is
substantial herniation pulmonary
hypoplasia is uncommon.
30. A 78 year old lady is admitted with small bowel obstruction, on
examination she has a distended
abdomen and the leg is held semi flexed. She has some groin pain
radiating to the ipsilateral knee.
Obturator hernia
The groin swelling in obturator hernia is subtle and hard to elicit
clinically.

Theme: Right iliac fossa pathology


A. Appendicitis
B. Mesenteric adenitis
C. Inflammatory bowel disease
D. Irritable bowel syndrome
E. Mesenteric cyst
F. Campylobacter infection
G. Appendix abscess
Please select the most likely diagnosis for the scenario given. Each
option may be used once, more than once or
not at all.
80. An 8 year old boy is examined by his doctor as part of a routine
clinical examination. The doctor
notices a smooth swelling in the right iliac fossa. It is mobile and the
patient is otherwise well.
Mesenteric cyst
Theme from April 2012 Exam
Mesenteric cysts are often smooth. Imaging with ultrasound and CT is
usually sufficient. Although
rare, they most often occur in young children (up to 30% present before
the age of 15). Many are
asymptomatic and discovered incidentally. Acute presentations are
recognised and may occur
following cyst torsion, infarction or rupture. Most cysts will be
surgically resected.
81. An 8 year old boy presents with abdominal pain,a twelve hour
history of vomiting, a fever of 38.3 oC
and four day history of diarrhoea. His abdominal pain has been present
for the past week.
You answered Campylobacter infection
The correct answer is Appendix abscess
The high fever and diarrhoea together with vomiting all point to a
pelvic abscess. The presence of
pelvic pus is highly irritant to the rectum, and many patients in this
situation will complain of
diarrhoea.
82. A 7 year old boy presents with a three day history of right iliac fossa
pain and fever. On examination
he has a temperature of 39.9o C. His abdomen is soft and mildly tender
in the right iliac fossa.
Mesenteric adenitis
High fever and mild abdominal signs in a younger child should raise
suspicion for mesenteric
adenitis. The condition may mimic appendicitis and many may require
surgery.

Theme: Management of pancreatic malignancy


A. Gastrojejunostomy
B. Pancreatoduodenectomy
C. MRI guided pancreatic stent
D. Endoscopic pancreatic stent
E. Duodenoduodenostomy
F. Pancreatic radiotherapy
Please select the most appropriate treatment for these patients with
pancreatic cancer. Each
option may be used once, more than once or not at all.
1. A 40 year old lady presents with new onset dyspepsia. She is
diagnosed as having a
localised cacinoma of the pancreatic head.
Pancreatoduodenectomy
Theme from January 2013 exam
Localised carcinoma of the pancreas is treated with a
pancreatoduodenectomy, the
eponymous name for this is a Whipples procedure. Newer variants of
the procedure
include pylorus preservation. Adjuvent chemotherapy is often used.
2. A 67 year old lady presents with jaundice and abdominal pain. Her
investigations show
a dilated common bile duct, a carcinoma of the pancreatic head
compressing the
pancreatic duct. Her liver contains bi-lobar metastasis.
Endoscopic pancreatic stent
Jaundice associated with pancreatic cancer is best managed with a stent.
These are
usually inserted at the time of ERCP. Consideration here should be
given to the use of a
metallic stent (which is contra indicated where resection is
contemplated).
3. A 67 year old lady presents with symptoms of persistent vomiting.
Her investigations
show gastric outlet obstruction from a carcinoma of the pancreatic head.
Her liver
contains bi-lobar metastases.
Gastrojejunostomy
Gastric outlet obstruction from pancreatic cancer is best managed with
a surgical bypass
procedure or a duodenal stent (if the disease is not resectable or
curable).
Theme: Gallstone disease
A. Uncomplicated biliary colic
B. Acute cholecystitis
C. Cholangitis
D. Gallbladder abscess
E. Acalculous cholecystitis
F. Pancreatitis
G. Gallstone ileus
Please select the most likely underlying diagnosis for the scenario given.
Each option may be
used once, more than once or not at all.
1. A 68 year old man with type 2 diabetes is admitted to hospital unwell.
On examination
he has features of septic shock and right upper quadrant tenderness. He
is not jaundiced.
Imaging shows a normal calibre bile duct and no stones in the
gallbladder.
Acalculous cholecystitis
Theme from April 2013 Exam
Acalculous cholecystitis is more common in patients with an underlying
co-morbidity.
The morbidity and mortality following intervention are higher than in
conventional
gallstone disease.
2. A 43 year old lady with know gallstones is admitted with a high fever
and jaundice. On
examination, she looks extremely unwell. Her abdomen is generally soft
although there
is some mild tenderness in the right upper quadrant.
Cholangitis
Features of jaundice, fever and systemic sepsis are typical of cholangitis.
3. A 34 year old lady is admitted with a 3 day history of colicky right
upper quadrant pain
which radiates to her back. The pain is now more constant. On
examination she is not
jaundiced, but has a temperature of 38.5oC. She has localised
peritonism in the right
upper quadrant.
Acute cholecystitis
The features of pain and fever with right upper quadrant pain are
suggestive of acute
cholecystitis. The short nature of the history makes an abscess less likely.

Theme: Management of jaundice


A. ERCP
B. MRCP
C. Percutaneous transhepatic cholangiogram
D. Laparotomy
E. Laparotomy and formation of hepatico-jejunostomy
F. Laparoscopic biliary bypass
G. CT scan
For each of the following scenarios please select the most appropriate
next stage of management.
Each option may be used once, more than once or not at all.
1. A 65 year old man is admitted with jaundice and investigations
demonstrate a
carcinoma of the pancreatic head. An ERCP is attempted but the
surgeon is unable to
cannulate the ampulla.
Percutaneous transhepatic cholangiogram
Theme from September 2012 Exam
Cancer of the pancreatic head will cause and obstructed jaundice and
intrahepatic duct
dilatation. When an ERCP has failed the most appropriate option is to
attempt a PTC.
This procedure is always preceded by an ultrasound (which presumably
this patient has
already had or they would not be undergoing an ERCP). Prior to
performing the PTC it
is important to stage the disease and establish resectability or not. This
is because the
PTC drains frequently dislodge and fall out. It is usually desirable to
pass a stent at the
time of doing the PTC to mitigate the effects of this problem.
2. A 48 year old lady is admitted with attacks of biliary colic and
investigations show
gallstones. A laparoscopic cholecystectomy is performed. The operation
is technically
challenging due to a large stone impacted in Hartmans pouch.
Following the operation
she fails to settle and becomes jaundiced and has bile draining into a
drain placed at the
surgical site.
ERCP
In this scenario it must be assumed that the bile duct has been damaged.
In most cases
an ERCP is the most appropriate investigation. This can also allow the
passage of a
stent if this is deemed to be safe and sensible.
3. A 34 year old lady is admitted with jaundice and undergoes an ERCP.
The procedure is
technically difficult and she is returned to the ward still jaundiced.
Unfortunately she
now has severe generalised abdominal pain.
You answered Laparotomy and formation of hepatico-jejunostomy
The correct answer is CT scan
There are two main differential diagnoses here. One is pancreatitis,
repeated trauma to
the ampulla and duct (if partially cannulated) is a major risk factor for
pancreatitis. The
second is the possibility that the duodenum has been perforated. ERCP
is performed
using a side viewing endoscope, the manipulation of which can be
technically
challenging for the inexperienced operator in a patient with abnormal
anatomy. A CT
scan is the best investigation to distinguish between these two
differential diagnoses.

Theme: Parotid gland disease


A. Pleomorphic adenoma
B. Adenoid cystic carcinoma
C. Sarcoid
D. Sjogrens syndrome
E. Sialolithiasis
F. Wharthins tumour
Please select the most likely explanation for the following patients with
parotid gland symptoms. Each option
may be used once, more than once or not at all.
1. A 50 year old female presents with bilateral parotid gland swelling
and symptoms of a dry mouth. On
examination she has bilateral facial nerve palsies. This improved
following steroid treatment.
You answered Sjogrens syndrome
The correct answer is Sarcoid
Theme from April 2013
Sarcoid occurs bilaterally in 70% of cases and facial nerve involvement
is recognised. Treatment is
conservative in most cases although individuals with facial nerve palsy
will usually receive steroids
with good effect.
2. A 50 year old women presents with a diffuse swelling in the region of
her right parotid together with
facial pain. On examination she has a right sided facial nerve palsy.
Adenoid cystic carcinoma
Adenoid cystic carcinoma commonly infiltrates the facial nerve and may
cause neuropathy and facial
pain.
3. A 50 year old lady presents with symptoms of a dry mouth that has
been present for the past few
months. She also has a sensation of grittiness in her eyes. On
examination she has a diffuse swelling of
her parotid gland. There is no evidence of facial nerve palsy.
You answered Pleomorphic adenoma
The correct answer is Sjogrens syndrome
Most patients with Sjogrens present in the post menopausal years. Multi
system involvement is common.
Theme: Disorders affecting the ear
A. Acoustic neuroma
B. Otosclerosis
C. Preauricular sinus
D. Acute suppurative otitis media
E. Cholesteatoma
F. Long standing perforation of the pars tensa
G. Otitis externa
Please select the most likely underlying explanation for the disorder
described. Each option may be used once,
more than once or not at all.
4. A 34 year old lady presents with a long standing offensive discharge
from the ear and on examination
is noted to have a reduction in her hearing of 40 decibels compared to
the opposite side.
You answered Long standing perforation of the pars tensa
The correct answer is Cholesteatoma
Theme from January 2013 Exam
The combination of offensive discharge and hearing loss is strongly
suggestive of cholesteatoma.
5. A 4 year old is brought to the general practitioner by her mother. She
has been distressed with ear pain
for the past 14 hours. She is constantly touching and pulling at her ear.
Whilst she is sat in the waiting
room her mother notices a discharge of foul smelling fluid from the ear,
following which the pain
resolves.
You answered Otitis externa
The correct answer is Acute suppurative otitis media
In young children acute suppurative otitis media is a common condition.
Rupture of the tympanic
membrane is a rare, but recognised complication.
6. A 4 year old child is brought to the clinic by his father. They are
concerned because the child has been
noted to have a small epithelial defect anterior to the left ear and is has
been noted to discharge foul
smelling material for the past 2 days.
Preauricular sinus
Pre auricular sinuses that a deeper may accumulate secretions and
produce foul smelling discharge.

Theme: Voice disorders


A. Vagus nerve injury
B. Thyroid nerve injury
C. Superior laryngeal nerve injury
D. Unilateral inferior laryngeal nerve injury
E. Bilateral inferior laryngeal nerves injuries
F. Stroke
G. Lacunar infarcts
H. None of the above
Please select the most likely reason for the scenarios given. Each option
may be used once, more than once or
not at all.
8. A 42 year old singer is admitted for a thyroidectomy. Post operatively
she is only able to make a
gargling noise. Her voice sounds breathy.
You answered Bilateral inferior laryngeal nerves injuries
The correct answer is Unilateral inferior laryngeal nerve injury
This patient has diplophonia which causes a gargling sound. This is
associated with dysphagia. This
can also be caused by a vagus nerve lesion, but the recurrent laryngeal
nerve is more at risk of
damage.
9. A 42 year old singer is admitted for a thyroidectomy. Post operatively
she is unable to sing high
pitched notes.
Superior laryngeal nerve injury
Theme from September 2011 Exam
SLN lesions cause difficulty in voice pitch.

Theme: Nasal diseases


A. Ethmoid sinus cancer
B. Maxillary sinus cancer
C. Ethmoid adenoma
D. Maxillary adenoma
E. Ethmoidal fracture
F. Nasal polyps
G. Sphenoid osteoma
H. Ethmoidal sinusitis
I. Maxillary sinusitis
Please select the most likely diagnosis for the scenario given. Each
option may be used once, more than once or
not at all.
11. A 56 year old man presents with symptoms of nasal pain, anosmia
and rhinorrhea. He has been well
until recently and has worked as a wood carver for many years.
You answered Ethmoid adenoma
The correct answer is Ethmoid sinus cancer
Theme from September 2012 Exam
Nasopharyngeal cancer is strongly associated with wood work. Most
cases require an occupational
exposure of greater than 10 years and are adenocarcinomas on histology.
Most cases are ethmoidal in origin (Hadfield E. Ann R Coll Surg Engl.
1970 June; 46(6): 301319)
12. A 32 year old female presents with recurrent episodes of
rhinorrhoea, the discharge is watery. She has
a medical history of asthma and intolerance of aspirin. On examination
she has multiple soft, semitransparent
polyps within her nasal cavity.
Nasal polyps
The combination of nasal polyps and atopy is well described. Some cases
will respond favourably to
systemic steroids and avoid surgery.
13. A child is brought to casualty complaining of a headache and a
sensation of pressure between the
eyes. On examination she is febrile with a smooth swelling overlying the
superomedial aspect of the
right eye. The eye is uncomfortable and there is a purulent discharge
from the inner canthus.
Ethmoidal sinusitis
Ethmoidal sinusitis may spread to the periorbital tissues resulting in
periorbital cellulitis. The
superomedial distribution makes a maxillary sinusitis less likely.

Theme: Parotid gland disorders


A. Sialectasis
B. Pleomorphic adenoma
C. Bacterial parotitis
D. Viral parotitis
E. Sjogren's syndrome
F. Adenoid cystic carcinoma
G. Mucoepidermoid carcinoma
H. Warthins tumour
Please select the most likely diagnosis for the scenario given. Each
option may be used once, more than once or
not at all.
16. A 40 year old lady with longstanding rheumatoid arthritis presents
with a dry mouth and parotid
gland swelling. Her symptoms transiently improved following
administration of a course of steroids
for her rheumatoid disease.
Sjogren's syndrome
Theme from April 2012 Exam
Sjogrens syndrome is linked to other autoimmune conditions such as
rheumatoid disease. A transient
improvement may be seen with steroid administration. Most patients
are treated with artificial saliva.
17. A 77 year old lady presents with facial pain and on examination is
found to have clinical evidence of
a facial nerve palsy and a parotid mass. Following surgical excision of
the lesion the histological
report comments on extensive perineural invasion.
Adenoid cystic carcinoma
Extensive perineural invasion is most commonly seen in patients with
adenoid cystic carcinoma. Both
Warthins tumours and mucoepidermoid carcinoma rarely show such
marked perineural infiltration.
18. An 18 year old boy presents with pancreatitis. He has bilateral
painful parotid enlargement.
Viral parotitis
In a young adult with parotid swelling and pancreatitis/orchitis/reduced
hearing/meningoencephalitis suspect mumps.

A 8 year old boy with recurrent attacks of otitis media is suspected of


developing a glue ear. If his sound
conduction is tested, which of the following is most consistent with a
unilateral middle ear effusion?
A. Negative Rinne's test on the ipsilateral side
B. Positive Rinne's test on the ipsilateral side
C. Positive Webers and Rinnes tests on the ipsilateral side
D. Positive Rinne's test on the contralateral side
E. Negative Webers test only on the contralateral side
Theme from 2008 Exam
Rinne's test will localise to the affected side (i.e. it is negative in
conductive deafness). In a positive Rinne's test
sound heard by air conduction is better than that conveyed by bone
conduction. Reduction of both air and bone
conduction in equal measure is a feature of sensorineural hearing loss.

Theme: Spinal disorders


A. Osteomyelitis
B. Potts disease of the spine
C. Scheuermanns disease
D. Transverse myelitis
E. Tabes dorsalis
F. Subacute degeneration of the cord
G. Brown-Sequard syndrome
H. Syringomyelia
I. Epidural haematoma
Which is the most likely diagnosis for the scenario given. Each option
may be used once, more than once or
not at all.
1. A 68 year old man presents to the plastics team with severe burns to
his hands. He is not distressed
by the burns. He has bilateral charcot joints. On examination there is
loss of pain and temperature
sensation of the upper limbs.
You answered Transverse myelitis
The correct answer is Syringomyelia
This patient has syringomyelia which selectively affects the
spinotholamic tracts.Syringomyelia is a
disorder in which a cystic cavity forms within the spinal cord. The
commonest variant is the Arnold-
Chiari malformation in which the cavity connects with a congenital
malformation affecting the
cerebellum. Acquired forms of the condition may occur as a result of
previous meningitis, surgery or
tumours. Many neurological manifestations have been reported,
although the classical variety spares
the dorsal columns and medial lemniscus and affecting only the
spinothalamic tract with loss of pain
and temperature sensation. The bilateral distribution of this patients
symptoms would therefore favor
syringomyelia over SCID or Brown Sequard syndrome. Osteomyelitis
would tend to present with
back pain and fever in addition to any neurological signs. Epidural
haematoma large enough to
produce neurological impairment will usually have motor symptoms in
addition to any selective
sensory loss, and the history is usually shorter.
2. A 24 year old man presents with localised spinal pain over 2 months
which is worsened on
movement. He is known to be an IVDU. He has no history suggestive of
tuberculosis. The pain is
now excruciating at rest and not improving with analgesia. He has a
temperature of 39 oC.
You answered Subacute degeneration of the cord
The correct answer is Osteomyelitis
In an IVDU with back pain and pyrexia have a high suspicion for
osteomylelitis. The most likely
organism is staph aureus and the cervical spine is the most common
region affected. TB tends to
affect the thoracic spine and in other causes of osteomyelitis the lumbar
spine is affected.
3. A 22 year man is shot in the back, in the lumbar region. He has
increased tone and hyper-reflexia of
his right leg. He cannot feel his left leg.
Brown-Sequard syndrome
Theme from January 2012 exam
Brown -Sequard syndrome is caused by hemisection of the spinal cord.
It may result from stab
injuries or lateral vertebral fractures. It results in ipsilateral paralysis
(pyramidal tract) , and also loss
of proprioception and fine discrimination (dorsal columns). Pain and
temperature sensation are lost
on the contra-lateral side. This is because the fibres of the spinothalamic
tract have decussated below
the level of the cord transection.

A 24 year old man is brought to the emergency department have


suffered a crush injury to his forearm.
Assessment demonstrates that the arm is tender, red and swollen. There
is clinical evidence of an ulnar fracture
and the patient cannot move their fingers. Which is the most
appropriate course of action?
A. Application of an external fixation device
B. Closed reduction
C. Debridement
D. Discharge and review in fracture clinic
E. Fasciotomy
Theme from April 2012
The combination of a crush injury, limb swelling and inability to move
digits should raise suspicion of a compartment syndrome that will
require a fasciotomy
A 10 year old boy presents with symptoms of right knee pain. The pain
has been present on most occasions for
the past three months and the pain typically lasts for several hours at a
time. On examination he walks with an
antalgic gait and has apparent right leg shortening. What is the most
likely diagnosis?
A. Perthes Disease
B. Osteosarcoma of the femur
C. Osteoarthritis of the hip
D. Transient synovitis of the hip
E. Torn medial meniscus

Theme from September 2012 Exam


There are many causes of the irritable hip in the 10-14 year age group.
Many of these may cause both hip pain
or knee pain. Transient synovitis of the hip the commonest disorder but
does not typically last for 3 months.
An osteosarcoma would not usually present with apparent limb
shortening unless pathological fracture had occurred. A slipped upper
femoral epiphysis can cause a similar presentation although it typically
presents later and with different patient characteristics.

An 8 year old child falls onto an outstretched hand and is brought to the
emergency department. He is
examined by a doctor and a bony injury is cleared clinically. He re-
presents a week later with pain in his hand.
What is the most likely underlying injury?
A. Fracture of the distal radius
B. Fracture of the scaphoid
C. Dislocation of the lunate
D. Rupture of flexor pollicis longus tendon
E. Bennett's fracture
Next question
Theme from January 2013 Exam
Scaphoid fractures in children will usually involve the distal pole and
are easily missed. The initial clinical
examination (and sometimes x-rays) may be normal and repeated
clinical examination and imaging is advised
for this reason. Whilst the other injuries may be sustained from a fall
onto an outstretched hand they are less
likely to be overlooked on clinical examination. In the case of a Bennetts
fracture, the injury mechanism is less
compatible with this type of injury.

Theme: Paediatric orthopaedics


A. Musculoskeletal pain
B. Congenital dysplasia of the hip
C. Slipped upper femoral epiphysis
D. Transient synovitis
E. Septic arthritis
F. Perthes disease
G. Tibial fracture
Please select the most likely diagnosis for the scenario given. Each
option may be used once, more than once
or not at all.
28. A 4 year boy presents with an abnormal gait. He has a history of
recent viral illness. His WCC is 11 and
ESR is 30.
Transient synovitis
Viral illnesses can be associated with transient synovitis. The WCC
should ideally be > 12 and the ESR
> 40 to suggest septic arthritis.
29. A 6 year old boy presents with an groin pain. He is known to be
disruptive in class. He reports that he is
bullied for being short. On examination he has an antalgic gait and pain
on internal rotation of the right
hip.
Perthes disease
This child is short, has hyperactivity (disruptive behaviour) and is
within the age range for Perthes
disease. Hyperactivity and short stature are associated with Perthes
disease.
30. An obese 12 year old boy is referred with pain in the left knee and
hip. On examination he has an
antaglic gait and limitation of internal rotation. His knee has normal
range of passive and active
movement.
Slipped upper femoral epiphysis
Similar theme to September 2012 Exam
Slipped upper femoral epiphysis is commonest in obese adolescent
males. The x-ray will show
displacement of the femoral epiphysis inferolaterally. Treatment is
usually with rest and non weight
bearing crutches.

A 19 year old soldier has just returned from a prolonged marching


exercise and presents with a sudden onset,
severe pain, in the forefoot. Clinical examination reveals tenderness
along the second metatarsal. Plain x-rays
are taken of the area, these demonstrate callus surrounding the shaft of
the second metatarsal. What is the most
likely diagnosis?
A. Stress fracture
B. Mortons neuroma
C. Osteochondroma
D. Acute osteomyelitis
E. Freiberg's disease

Theme from 2011 Exam


A short history of pain together with clinical examination and
radiological signs affecting the second
metatarsal favour a stress fracture. The fact that callus is present
suggests that immobilisation is unlikely to be
beneficial. Freibergs disease is an anterior metatarsalgia affecting the
head of the second metarsal, it typically
occurs in the pubertal growth spurt. The initial injury was thought to be
due to stress microfractures at the
growth plate. The key feature in the history which distinguishes the
injury as being stress fracture is the
radiology. In Freibergs disease the x-ray changes include; joint space
widening, formation of bony spurs,
sclerosis and flattening of the metatarsal head.

Theme: Diseases affecting the spine


A. Spondylolysis
B. Spina bifida occulta
C. Spondylolisthesis
D. Meningomyelocele
E. Meningocele
F. Scoliosis - non structural
G. Scoliosis
H. Ankylosing spondylitis
I. Scheuermann's disease
Please select the most likely underlying diagnosis for the condition
described. Each condition may be used
once, more than once or not at all.
39. A 19 year old female is involved in an athletics event. She has just
completed the high jump when she
suddenly develops severe back pain and weakness affecting both her
legs. on examination she has a
prominent sacrum and her lower back is painful.
Spondylolisthesis
Theme from September 2012 Exam
Young athletic females are the group most frequently affected by
spondylolythesis who have a
background of spondylolysis. Whilst the latter condition is a risk factor
for spondylolythesis the former
condition is most likely in a young athletic female who presents with
sudden pain.
40. A 15 year old boy is brought to the clinic by his mother who is
concerned that he has a mark overlying
his lower spine. On examination the boy has a patch of hair overlying
his lower lumbar spine and a
birth mark at the same location. Lower limb neurological examination
is normal.
Spina bifida occulta
Spina bifida occulta is a common condition and may affect up to 10% of
the population. The more
severe types of spina bifida have more characteristic skin changes.
Occasionally the unwary surgeon is
persuaded to operate on these "cutaneous" changes and we would
advocate performing an MRI scan
prior to any such surgical procedure in this region.
41. A 19 year old female presents to the clinic with progressive pain in
her neck and back. The condition
has been progressively worsening over the past 6 months. She has not
presented previously because she
was an inpatient with a disease flare of ulcerative colitis. On
examination she has a stiff back with
limited spinal extension on bending forwards.
You answered Spondylolysis
The correct answer is Ankylosing spondylitis
Ankylosing spondylitis is associated with HLA B27, there is a strong
association with ulcerative colitis
in such individuals. The clinical findings are usually of a kyphosis
affecting the cervical and thoracic
spine. Considerable symptomatic benefit may be obtained using non
steroidal anti inflammatory drugs.
These should be used carefully in patients with inflammatory bowel
disease who may be taking
steroids.

Theme: Knee injuries


A. Anterior cruciate ligament rupture
B. Posterior cruciate ligament rupture
C. Medial collateral ligament tear
D. Lateral collateral ligament tear
E. Torn meniscus
F. Chondromalacia patellae
G. Dislocated patella
H. Fractured patella
I. Tibial plateau fracture
What is the most likely injury for scenario given? Each option may be
used once, more than once or not at all.
52. A 38 year old man is playing football when he slips over during a
tackle. His knee is painful
immediately following the fall. Several hours later he notices that the
knee has become swollen.
Following a course of non steroidal anti inflammatory drugs and rest
the situation improves. However,
complains of recurrent pain. On assessment in clinic you notice that it is
impossible to fully extend the
knee, although the patient is able to do so when asked.
You answered Tibial plateau fracture
The correct answer is Torn meniscus
Theme from September 2012 Exam
Twisting sporting injuries followed by delayed onset of knee swelling
and locking are strongly
suggestive of a menisceal tear. Arthroscopic menisectomy is the usual
treatment.
53. A 34 year old woman is a passenger in a car during an accident. Her
knee hits the dashboard. On
examination the tibia looks posterior compared to the non injured knee.
You answered Torn meniscus
The correct answer is Posterior cruciate ligament rupture
In ruptured posterior cruciate ligament the tibia lies back on the femur
and can be drawn forward during
a paradoxical draw test.
54. A 28 year old professional footballer is admitted to the emergency
department. During a tackle he is
twisted with his knee flexed. He hears a loud crack and his knee rapidly
becomes swollen.
You answered Fractured patella
The correct answer is Anterior cruciate ligament rupture
This is common in footballers as the football boot studs stick to the
ground and high twisting force is
applied to a flexed knee. Rapid joint swelling also supports the diagnosis.

Theme: Ankle fractures


A. Surgical fixation
B. Below knee amputation
C. Aircast boot
D. Application of full leg plaster cast to include midfoot
E. Application of below knee plaster cast to include the midfoot
F. Application of external fixation device
G. Application of compression bandage and physiotherapy.
Please select the most appropriate management for the injury type
described. Each option may be used once,
more than once or not at all.
65. A 24 year old man falls sustaining an inversion injury to his ankle.
On examination he is tender over the
lateral malleolus only. On x-ray there is a fibular fracture that is distal
to the syndesmosis.
Application of below knee plaster cast to include the midfoot
Theme from 2008 Exam
These distal injuries are generally managed conservatively.
Conservative management will involve a
below knee cast, this will need to extend to the midfoot. It can be
substituted for an aircast boot once
radiological union is achieved.
66. An 86 year old lady stumbles and falls whilst opening her front door.
On examination her ankle is
swollen with both medial and lateral tenderness. X rays demonstrate a
fibular fracture at the level of the
syndesmosis.
Application of below knee plaster cast to include the midfoot
Although, this is a potentially unstable injury operative fixation in this
age group generally gives poor
results owing to poor quality bone. A below knee cast should be applied
in the first instance. If this fails
to provide adequate control it can be extended above the knee.
67. A 25 year old man suffers an injury whilst playing rugby involving a
violent twist to his left lower leg.
On examination both malleoli are tender and the ankle joint is very
swollen. On x-ray there is a spiral
fracture of the fibula and widening of the ankle mortise.
You answered Application of external fixation device
The correct answer is Surgical fixation
This is a variant of the Weber C fracture in which disruption of the
tibio-fibular syndesmosis occurs
leading to joint disruption. Surgical repair is warranted.

Theme: Fracture management


A. Application of external fixator
B. Open reduction and internal fixation
C. Fasciotomy
D. Skeletal traction
For the following upper limb injuries please select the most appropriate
initial management. Each option may
be used once, more than once or not at all.
78. A 32 year old man falls from a ladder and sustains a fracture of his
proximal radius. On examination he
has severe pain in his forearm and diminished distal sensation. There is
a single puncture wound present
at the fracture site.
Fasciotomy
Theme from April 2012 Exam
Pain and neurological symptoms in a tight fascial compartment coupled
with a high velocity injury
carry a high risk of compartment syndrome and prompt fasciotomy
should be performed.
79. A 32 year old man falls a sustains a fracture of his distal humerus.
The fracture segment is markedly
angulated and unstable. There is a puncture site overlying the fracture
site.
You answered Open reduction and internal fixation
The correct answer is Application of external fixator
Wide exposure to plate the humerus is generally inadvisable owing to its
many important anatomical
relations. Both intramedullary nailing and external fixation are
reasonable treatments. However, in the
presence of an open fracture application of an external fixator and
appropriate tissue debridement would
be most appropriate.
80. A 24 year old man sustains a distal radius fracture during a game of
rugby. Imaging shows a
comminuted fracture with involvement of the articular surface.
You answered Application of external fixator
The correct answer is Open reduction and internal fixation
Meticulous anatomical alignment of the fracture segments is crucial to
avoid the development of
osteoarthritis and risk of malunion.

Theme: Paediatric gastrointestinal disorders


A. Duodenal atresia
B. Hypertrophy of the pyloric sphincter
C. Budd Chiari Syndrome
D. Intussceception
E. Oesophageal atresia
F. Congenital diaphragmatic hernia
G. Cystic fibrosis
H. Intestinal malrotation
I. Gastroenteritis
Please select the most likely diagnosis for the scenario given. Each
option may be used once, more than once or not at all.
1. A two week old child is brought to the emergency department by his
parents. He was slow to establish on feeds but
was discharged home three days following delivery. During the past 7
hours he has been vomiting and the vomit is
largely bile stained. On examination he has a soft, scaphoid abdomen.
You answered Duodenal atresia
The correct answer is Intestinal malrotation
Theme from April 2013 Exam
The combination of scaphoid abdomen and bilious vomiting is highly
suggestive of intestinal malrotation and
volvulus. An urgent upper GI contrast study and ultrasound is required.
2. A 4 week old baby is developing well and develops profuse and
projectile vomiting after feeds. He has been losing
weight and the vomit is described as being non bilious.
Hypertrophy of the pyloric sphincter
A history of projective vomiting and weight loss is a common story
suggestive of pyloric stenosis. The vomit is often
not bile stained. Diagnosis is further suggested by hyperchloraemic
metabolic alkalosis and a palpable tumour on test
feeding.
3. A 1 day old child is born by emergency cesarean section for foetal
distress. On examination he has decreased air entry
on the left side of his chest and a displaced apex beat. Abdominal
examination demonstrates a scaphoid abdomen but
is otherwise unremarkable.
Congenital diaphragmatic hernia
Displaced apex beat and decreased air entry are suggestive of
diaphragmatic hernia. The abdomen may well be
scaphoid in some cases. The underlying lung may be hypoplastic and
this correlates directly with prognosis.

A 2-day-old baby girl is noted to become cyanotic whilst feeding and


crying. A diagnosis of congenital heart disease is
suspected. What is the most likely cause?
A. Transposition of the great arteries
B. Coarctation of the aorta
C. Patent ductus arteriosus
D. Tetralogy of Fallot
E. Ventricular septal defect
Congenital heart disease
cyanotic: TGA most common at birth, Fallot's
most common overall
acyanotic: VSD most common cause
Theme from April 2011 exam

A 6 month old boy is brought to the clinic by his mother. She is


concerned that his testes are not located into the scrotum. She
has noticed them only when he is in the bath, but not at any other time.
What is the most likely underlying diagnosis?
A. Rectractile testis
B. Ectopic testis
C. Undescended testis
D. Testicular agenesis
E. Intersex child
Theme from April 2012 Exam
A testis that appears in warm conditions or which can be brought down
on clinical examination and does not immediately
retract is usually a retractile testis.

Theme: Skin disease


A. Squamous cell carcinoma
B. Bowens disease
C. Actinic keratosis
D. Basal cell carcinoma
E. Malignant melanoma
F. Keratoacanthoma
G. Apthous ulcer
H. Pyogenic granuloma
Please select the most likely underlying diagnosis for each of the
following skin lesions. Each option may be
used once, more than once or not at all.
1. A 53 year old man presents with a nodule on his chin. He is concerned
because it has grown extremely
rapidly over the course of the preceding week. On examination he has a
swollen, red, dome shaped
lesion with a central defect that contains a keratinous type material.
Keratoacanthoma
Theme from April 2013 Exam
Keratoacanthomas are characterised by a rapid growth phase. This
may mimic amelanotic melanoma
(although such rapid growth is rare even in these lesions). The keratin
core is the clue as to the true
nature of the lesion.
2. A 68 year old farmer presents with a skin lesion on his forehead. It
has been present for the past 6
months and has grown slightly in size during that time. On examination
he has an ulcerated lesion with
pearly white raised edges that measures 2cm in diameter.
Basal cell carcinoma
The raised pearly edges in an ulcerated lesion at a sun exposed site
makes BCC most likely.
3. A 34 year old gardener presents with a lesion affecting the dorsum of
his right hand. It has been
present for the past 10 days and occurred after he had been pruning
rose bushes. On examination he
has a raised ulcerated lesion which bleeds easily on contact.
Pyogenic granuloma
Trauma is a common precipitant of pyogenic granuloma and contact
bleeding and ulceration are
common.

Theme: Skin disorders


A. Basal cell carcinoma
B. Dermatofibroma
C. Pilar cyst
D. Epidermoid cyst
E. Spitz naevus
F. Seborrhoeic keratosis
G. Atypical naevus
H. Capillary cavernous haemangioma
Please select the most likely underlying nature of the skin lesion
described. Each option may be used once,
more than once or not at all.
4. A 70 year old lady presents with a number of skin lesions that she
describes as unsightly. On
examination she has a number of raised lesions with a greasy surface
located over her trunk. Apart
from having a greasy surface the the lesions also seem to have scattered
keratin plugs located within
them.
Seborrhoeic keratosis
Theme from September 2012 Exam
Seborrhoeic keratosis may have a number of appearances. However, the
scaly, thick, greasy surface
with scattered keratin plugs makes this the most likely diagnosis.
5. A 28 year old female presents with a small nodule located on the back
of her neck. It is excised for
cosmetic reasons and the histology report states that the lesion consists
of a sebum filled lesion
surrounded by the outer root sheath of a hair follicle.
You answered Epidermoid cyst
The correct answer is Pilar cyst
Pilar cysts may contain foul smelling cheesy material and are
surrounded by the outer part of a hair
follicle. Because of their histological appearances they are more
correctly termed pilar cysts than
sebaceous cysts.
6. A 21 year old lady presents with a nodule on the posterior aspect of
her right calf. It has been present
at the site for the past 6 months and occurred at the site of a previous
insect bite. Although the nodule
appears small, on palpation it appears to be nearly twice the size it
appears on examination. The
overlying skin is faintly pigmented.
You answered Spitz naevus
The correct answer is Dermatofibroma
Dermatofibromas may be pigmented and are often larger than they
appear. They frequently occur at
sites of previous trauma.
A 22 year old man presents with an infected sebaceous cyst. The cyst
itself is swollen, discharging pus and has
some surrounding erythema. What is the most appropriate treatment?
A. Excision of the cyst of closure of the defect with interrupted 3/0 silk
B. Excision of the cyst and closure of the defect with subcuticular 4/0
undyed nylon
C. Incision and drainage with excision of the cyst wall and packing of
the defect
D. Incision and drainage with conservation of the cyst wall and packing
of the defect
E. Administration of oral co-amoxyclav and definitive surgery once the
infection has cleared
Similar theme in January 2013 Exam
The correct treatment for an infected sebaceous cyst is incision and
drainage with removal of the cyst wall.
Conservation of the cyst wall will invariably lead to recurrence. Under
no circumstances should an infected
wound like this be primarily closed. The administration of antibiotics
without drainage of sepsis is futile.

Theme: Dermatological manifestations of disease


A. Pyoderma gangrenosum
B. Erythroderma
C. Dermatitis herpetiformis
D. Acanthosis nigricans
E. Multiple lipomata
F. Multiple neurofibromata
G. Multiple telangectasia
H. None of the above
Please select the skin disease associated with the condition described.
Each option may be used once, more than
once or not at all.
14. A 22 year old man is investigated for weight loss. A duodenal biopsy
taken as part of his
investigations shows total villous atrophy and lymphocytic infiltrate. He
has a skin lesion that has
small itchy papules.
You answered Erythroderma
The correct answer is Dermatitis herpetiformis
Theme from September 2012 Exam
The patient has coeliac disease and this is associated with dermatitis
herpetiformis.
15. A 72 year old man is investigated for weight loss. On examination he
is deeply jaundiced and
cachectic. He also has a dark velvety lesion coating his tongue.
Acanthosis nigricans
Acanthosis nigricans may be associated with GI malignancies such as
gastric and pancreatic cancer.
16. A lesion that may occur in a 32 year old man with long standing
Crohns disease.
Pyoderma gangrenosum
Pyoderma gangrenosum may occur in Crohns disease.

A 21 year old man is involved in a road traffic accident. After a


transient period of concussion he is found to
have a GCS of 15 by the paramedics. On arrival at hospital he is
monitored in a side room of the emergency
department. When he is next observed he is noted to have a GCS of 3
and a blown right pupil. Which of the
processes below best accounts for this deterioration?
A. Hydrocephalus
B. Intraventricular bleed
C. Sub dural bleed
D. Trans tentorial herniation
E. Sub arachnoid haemorrhage
Theme from April 2012 Exam
The presence of a blown right pupil is a sign of a third cranial nerve
compression. The most likely cause is an
extradural bleed. However, since this option is not listed the process of
trans tentorial herniation would be the
most applicable answer. Intraventricular bleeds are typically more
common in premature neonates, deterioration
due to hydrocephalus is more chronic.
Theme: Intracranial bleeds
A. Intraventricular haemorrhage
B. Chronic sub dural bleed
C. Acute sub dural bleed
D. Extra dural haemorrhage
E. Sub arachnoid haemorrhage
Please select the most likely underlying diagnosis for the scenario
described. Each option may be used once,
more than once or not at all.
2. A 50 year old alcoholic man attends the emergency department. His
main reason for presenting is that he
has no home to go to. On examination he has no evidence of involvement
in recent trauma, a skull x-ray
fails to show any evidence of skull fracture. He is admitted and twelve
hours following admission he
develops sudden onset headache, becomes comatose and then dies.
You answered Acute sub dural bleed
The correct answer is Sub arachnoid haemorrhage
Theme from April 2013 Exam
The absence of trauma here makes an acute sub dural and extra dural
bleed unlikely. Chronic sub dural
bleeds would usually cause a more gradual deterioration than is seen
here. The absence of any skull
fracture also makes an underlying intra cranial bleed less likely. Sudden
onset headaches, together with
sudden deterioration in neurological function are typical of a sub
arachnoid haemorrhage.
3. A 50 year old lady is admitted having fallen down some stairs
sustaining multiple rib fractures 36 hours
previously. On examination she is confused and agitated and has clinical
evidence of lateralising signs.
She deteriorates further and then dies with no response to resuscitation.
You answered Extra dural haemorrhage
The correct answer is Acute sub dural bleed
The time frame of deterioration of an acute sub dural bleed would fit
with this scenario. They are highly
lethal and not uncommon injuries. As the bleed enlarges, lateralising
signs may be seen and eventually
coning and death will occur.
4. A male infant is born at 28 weeks gestation by emergency cesarean
section. He is taken to theatre for a
colostomy due to an imperforate anus. He initially seems to be
progressing well. However, he begins to
develop decerebrate posturing and is becoming increasingly obtunded.
Intraventricular haemorrhage
Acute neurological deterioration in premature neonates is usually due
to intraventricular haemorrhage.
Diagnosis is made by cranial ultrasound. Development of hydrocephalus
may necessitate surgery.

Theme: Head injury- assessment


A. 3
B. 4
C. 6
D. 8
E. 10
F. 15
G. 12
H. 13
I. 5
J. 7
For each of the scenarios given please determine the most likely
Glasgow Coma Score. Each option may be
used once, more than once or not at all.
8. A 20 year old man is hit over the head with a mallet. On arrival in the
accident and emergency
department he opens his eyes to pain and groans or grunts. On
application of a painful stimulus to his
hands, he extends his arm at the elbow.
You answered 8
The correct answer is 6
Theme from 2011 Exam
Theme from September 2012 Exam
E=2, V= 2, M=2.
9. A 20 year old man falls over and bangs his head whilst intoxicated.
On arrival in the emergency
department he opens his eyes in response to speech, and is able to speak,
although he is disorientated.
He obeys motor commands.
You answered 12
The correct answer is 13
E=3, V=4, M=6.
10. A 20 year old man is hit over the head with an iron bar. On arrival
in the emergency department he
opens his eyes in response to pain, his only verbal responses are in the
form of groans and grunts. On
application of a painful stimulus to his hands, he flexes his forearms
away from the painful stimuli.
You answered 7
The correct answer is 8
E=2, V=2, M=4.
Appropriate flexion to pain carries a higher score than decorticate
posturing or inappropriate flexion.

A 33 year old lady develops a thunderclap headache and collapses. A


CT scan shows that she has developed a
subarachnoid haemorrhage. She currently has no evidence of raised
intracranial pressure. Which of the
following drugs should be administered?
A. None
B. Atenotol
C. Labetolol
D. Nimodipine
E. Mannitol
Theme from 2007 Exam
Nimodipine is a calcium channel blocker. It reduces cerebral vasospasm
and improves outcomes. It is
administered to most cases of sub arachnoid haemorrhage.
A 28 year old women collapses and is found to be deeply comatose with
a GCS of 3. She is admitted to
hospital, intubated and ventilated. A CT scan shows an extensive sub
arachnoid haemorrhage. A urinary
catheter is inserted. What type of urine output would be expected?
A. 30ml concentrated urine per hour
B. 30ml diluted urine per hour
C. 60ml diluted urine per hour
D. 300ml concentrated urine per hour
E. 300ml diluted urine per hour
Theme from 2011 Exam
An extensive CNS insult is likely to result in centrally mediated diabetes
insipidus. This may either be the result
of hypothalamic injury directly or the result of disruption of the
hypothalamic-pituitary connections. The result
is a large volume diuresis with diluted urine.

Theme: Visual defects


A. Right homonymous hemianopia
B. Left homonymous hemianopia
C. Right superior quadranopia
D. Left superior quadranopia
E. Right inferior quadranopia
F. Left inferior quadranopia
G. Upper bitemporal hemianopia
H. Lower bitemporal hemianopia
What is the most likely visual field defect for the scenario given? Each
option may be used once, more than
once or not at all.
19. A 42 year old woman is admitted to the vascular ward for an
endarterectomy. Her CT report confirms
a left temporal lobe infarct.
You answered Right homonymous hemianopia
The correct answer is Right superior quadranopia
Temporal lesions cause a contralateral superior quadranopia. Think
temporal area is at the top of the
head i.e. superior quadranopia.
20. A 22 year old man is referred to urology with possible urinary
retention. He is passing huge amounts
of urine. Post void bladder ultrasound is normal.
You answered Upper bitemporal hemianopia
The correct answer is Lower bitemporal hemianopia
Theme from January 2012 exam
This patient has diabetes insipidus due to a craniopharyngioma. This
causes a lower bitemporal
hemianopia.
21. A 53 year old man is admitted to the vascular ward for a carotid
endarterectomy. His CT head report
confirms a left parietal lobe infarct.
You answered Right superior quadranopia
The correct answer is Right inferior quadranopia
Parietal lesions cause a contralateral inferior quadranopia.
Superior quadranopia = temporal lobe lesion
Inferior quadranopia = parietal lobe lesion

Theme: Transplantation
A. Xenograft
B. Allograft
C. Autograft
D. Isograft
Please select the type of transplantation that has occurred in the
situation described. Each option may be used
once, more than once or not at all.
1. A 38 year old lady donates her kidney to her identical twin sibling.
Isograft
Theme from January 2013
Identical twin- twin donations are usually genetically identical and are
therefore isografts.
2. A 53 year old man with severe angina undergoes a coronary artery
bypass procedure and his long
saphenous vein is used as a bypass conduit.
Autograft
The long saphenous vein is one of the commonest autografts in surgery.
3. A 38 year old lady donates her kidney to her niece.
Allograft
Though related this donor will not be genetically identical and thus this
will be an allograft.

Theme: Renal transplant complications


A. Acute tubular necrosis
B. Renal artery thrombosis
C. Bladder occlusion
D. Ureteric occlusion
E. Acute rejection
F. Acute on chronic rejection
G. Hyperacute rejection
For each of the scenarios given please select the most likely underlying
process from the list below. Each option
may be used once, more than once or not at all.
5. A 45 year old man with end stage renal failure undergoes a cadaveric
renal transplant. The transplanted
organ has a cold ischaemic time of 26 hours and a warm ischaemic time
of 54 minutes. Post
operatively the patient receives immunosuppressive therapy. Ten days
later the patient has gained
weight, becomes oliguric and feels systemically unwell. He also
complains of swelling over the
transplant site that is painful.
Acute rejection
Theme from April 2012 Exam
The features described are those of worsening graft function and acute
rejection. The fact that there is a
10 day delay goes against hyperacute rejection. Cold ischaemic times
are a major factor for delayed
graft function. However, even 26 hours is not incompatible with graft
survival.
6. A 44 year old man with end stage renal failure undergoes a live donor
renal transplant. During the
immediate post operative period a good urine output is recorded.
However, on return to the ward the
nursing staff notice that the urinary catheter is no longer draining.
However, the urostomy is
continuing to drain urine.
You answered Ureteric occlusion
The correct answer is Bladder occlusion
The most likely explanation for this event is a blocked catheter. This
may be the result of blood clot
from the ureteric anastomosis. Bladder irrigation will usually resolve
the problem.
7. A 43 year old man undergoes a live donor renal transplant. The
donor's right kidney is anastomosed to
the recipient. On removal of the arterial clamps there is good urinary
flow noted and the wounds are
closed. On return to the ward the nurses notice that the patient
suddenly becomes anuric and irrigation
of the bladder does not improve the situation.
You answered Acute tubular necrosis
The correct answer is Renal artery thrombosis
Right sided live donor transplants are extremely rare. This is because
the vena cava precludes
mobilisation of the right renal artery. The short right renal artery that
is produced therefore presents a
major challenge. The sudden cessation of urine output in this context is
highly suggestive of an acute
thrombosis. Delay in thrombectomy beyond 1 hour almost inevitably
results in graft loss.

A 49 year old female is due to undergo a renal transplant. Apart from


ABO compatibility which of the
following is most important in matching donor and recipient organs?
A. HLA-DR
B. Rhesus
C. HLA- A
D. HLA-B
E. Kidd
Theme from January 2013 Exam
The effect of HLA-DR mismatches are the most clinically significant,
since HLA-DR mismatch increases graft
loss five fold. HLA-B increases graft loss three fold and HLA-A
increases the risk two fold. Rhesus is not used
to match organs to recipients. Kidd is a minor group and of no
significance.

Theme: Causes of dysphagia


A. Benign oesophageal stricture
B. Globus
C. Carcinoma of the oesophagus
D. Achalasia
E. Hiatus hernia
F. Pharyngeal pouch
G. Oesophageal web
H. Oesophageal atresia
Please select the the most likely cause of swallowing difficulties for the
scenarios described.
Each option may be used once, more than once or not at all.
1. A 55 year old man presents with a history of progressive dysphagia
over the past 4
weeks. For the preceding 5 years he had regularly attended his general
practitioner with
symptoms of dyspepsia and reflux.
Carcinoma of the oesophagus
Theme from January 2013 Exam
A short history of progressive dysphagia in a middle aged man who has
a background
history of reflux is strongly suggestive of malignancy. Long standing
reflux symptoms
may be suggestive of a increased risk of developing Barretts oesophagus.
Note that not
all patients with Barretts transformation alone are symptomatic.
2. A 40 year old man presents with symptoms of dysphagia that have
been present for
many months. His investigations demonstrate lack of relaxation of the
lower
oesophageal sphincter during swallowing.
Achalasia
Patients with dysphagia will usually undergo an upper GI endoscopy as
a first line
investigation. Where this investigation is normal, the next stage is to
perform studies
assessing oesophageal motility. These comprise fluroscopic barium
swallows and
oesophageal manometry and pH studies. Lack of sphincter relaxation
suggests achalasia
(pressures are usually high).
3. A 4 year old presents with sudden onset of dysphagia. He undergoes
an upper GI
endoscopy and a large bolus of food is identified in the mid oesophagus.
He has no
significant history, other than a tracheo-oesophageal fistula repair soon
after birth.
You answered Hiatus hernia
The correct answer is Benign oesophageal stricture
Children with tracheo-oesophageal fistulas will commonly develop
oesophageal
strictures following repair. These may require regular dilations
throughout childhood.

A 45 year old man undergoes an upper gastrointestinal endoscopy for a


benign oesophageal
stricture. This is dilated and he suffers an iatrogenic perforation at the
site. His imaging shows a
small contained leak and a small amount of surgical emphysema. What
is the most appropriate
nutritional option?
A. Nil by mouth and intravenous fluids alone
B. Intravenous fluids and sips orally
C. Total parenteral nutrition
D. Nasogastric feeding
E. PEG tube feeding
Theme from April 2013 Exam
Iatrogenic perforations of the oesophagus may be managed non
operatively. This usually
involves a nil by mouth regime, tube thoracostomy may be needed.
Total parenteral nutrition is
the safest option. Insertion of NG feeding tubes and PEG tubes may
complicate the process or
allow feed to enter the perforation site.

A 58 year old lady has a two year history of worsening dysphagia. In


addition to this she has a
nocturnal cough. Over this time she has lost a total of 8kg in weight.
Which of the processes
below is the most likely explanation for this?
A. Gastro-oesophageal reflux disease
B. Barretts oesophagus
C. Carcinoma
D. Mallory Weiss tear
E. Achalasia
Theme from April 2013 Exam
Such marked weight loss should arouse suspicion of cancer. She is most
likely to have a
longstanding stricture associated with oesophagitis and Barretts
oesophagus. This may progress
to carcinoma which will tend to occur in association with worsening
dysphagia and weight loss.

Theme: Oesophageal disease


A. Oesophagectomy
B. Endoscopic sub mucosal dissection
C. Photodynamic therapy
D. Insertion of oesophageal stent
E. Chemotherapy
F. Radiotherapy
Please select the most appropriate treatment modality for the scenario
given. Each option may be
used once, more than once or not at all.
11. A 52 year old man with long standing Barretts oesophagus is
diagnosed with high
grade dysplasia on recent endoscopy. The lesions are multifocal and
mainly distally
sited.
Oesophagectomy
Similar theme in 2011 exams
Some may argue for local therapy. However, in young patients who are
otherwise fit,
multifocal disease such as this should probably be resected.
12. A 72 year old man presents with dysphagia. He is investigated and
found to have an
adenocarcinoma of the distal oesophagus. His staging investigations
have revealed a
solitary metastatic lesion in the right lobe of his liver.
You answered Chemotherapy
The correct answer is Insertion of oesophageal stent
Similar theme in 2011 exam
Although he may be palliated with chemotherapy a stent will produce
the quickest
clinical response. Metastatic disease is usually a contra indication to
oesophageal
resection.
13. An 83 year old lady with long standing Barretts oesophagus is
diagnosed with a 1cm
focus of high grade dysplasia 3cm from the gastrooesophageal junction.
Endoscopic sub mucosal dissection
As she is elderly and the disease localised EMR is an appropriate first
line step.
The technique involves raising the mucosa containing the lesion and
then using an
endoscopic snare to remove it. This technique is therefore minimally
invasive.
However, it is only suitable for early superficial lesions. Deeper invasion
would carry
a high risk of recurrence.
Theme: Dysphagia
A. Mallory Weiss Tear
B. Hiatus hernia
C. Squamous cell carcinoma of the oesophagus
D. Adenocarcinoma of the oesophagus
E. Peptic stricture
F. Motility disorder
For each of the following scenarios please select the most likely
underlying disorder. Each
option may be used once, more than once or not at all.
28. A 56 year old lady presents with a 6 month history of dysphagia to
solids. She has a
long history of retrosternal chest pain that is worse on lying flat and
bending forwards.
She undergoes an upper GI endoscopy where a smooth stricture is
identified.
Peptic stricture
Theme from April 2012 Exam
A six month history of dysphagia is a relatively long history and makes
malignancy
less likely. The lesion should be biopsied for histological confirmation.
Long standing
oesophagitis may be complicated by the development of strictures,
Barretts
oesophagus or both.
29. A 76 year old man presents with a 5 week history of progressive
dysphagia. An upper
GI endoscopy is performed and the surgeon notices changes that are
compatible with
Barretts oesophagus. The oesophagus is filled with food debris that
cannot be cleared
and the endoscope encounters a resistance that cannot be passed.
Adenocarcinoma of the oesophagus
A short history of dysphagia together with food debris and Barretts
changes makes
adenocarcinoma the most likely diagnosis.
30. A 22 year old man presents with a 5 month history of episodic
retrosternal chest pain
together with episodes of dysphagia to liquids. An upper GI endoscopy
is performed
and no mucosal abnormality is seen.
Motility disorder
Dysphagia that is episodic and varies between solids and liquids is more
likely to
represent a motility disorder.

Theme: Management of urinary tract calculi


A. Nephrectomy
B. Open ureteric exploration
C. Extra corporeal shock wave lithotripsy
D. Percutaneous nephrostomy
E. Pyeloplasty
F. Conservative management
G. Percutaneous nephrolithotomy
Please select the most appropriate management for the scenario given.
Each option may be used once, more
than once or not at all.
1. A 23 year old male is admitted with left sided loin pain and fever. His
investigations demonstrate a left
sided ureteric calculi that measures 0.7cm in diameter and associated
hydronephrosis.
You answered Percutaneous nephrolithotomy
The correct answer is Percutaneous nephrostomy
Theme from January 2013 Exam
An obstructed, infected system is an indication for urgent
decompression. This may be achieved by
ureteroscopy or nephrostomy. In addition to this the patient should also
receive broad spectrum,
intravenous antibiotics.
2. A 23 year old man is admitted with left sided loin pain that radiates to
his groin. His investigations
demonstrate a 1cm left sided ureteric calculus with no associated
hydronephrosis.
Extra corporeal shock wave lithotripsy
Stones with a total volume of less than 2cm can be considered for
lithotripsy. If it is impacted in the
upper ureter then some may consider a ureteroscopy.
3. A 30 year old male presents with left sided loin pain. His
investigations demonstrate a large left sided
staghorn calculus that measures 2.3cm in diameter.
You answered Extra corporeal shock wave lithotripsy
The correct answer is Percutaneous nephrolithotomy
Large, proximal stones are generally best managed with a percutaneous
nephrolithotomy. The use of
lithotripsy has low clearance rates. Where stones remain after the initial
procedure a repeat
percutaneous nephrolithotomy is generally preferred over follow up
lithotripsy.

Theme: Urinary tract trauma


A. Urinary tract infection
B. Bladder outlet obstruction
C. Bulbar urethral rupture
D. Membranous urethral rupture
E. Bladder rupture
F. Bladder contusion
For the scenario given please select the most likely injury. Each option
may be used once, more than once or not
at all.
7. A 56 year old man is involved in a road traffic accident. He is found
to have a pelvic fracture. He
reports that he has some lower abdominal pain. He has peritonism in
the lower abdomen. The nursing
staff report that he has not passed any urine. A CT scan shows evidence
of free fluid.
Bladder rupture
A pelvic fracture and lower abdominal peritonism should raise
suspicions of bladder rupture
(especially as this man cannot pass urine).
8. A 52 year old man falls off his bike. He is found to have a pelvic
fracture. On examination he is found
to have perineal oedema and on PR the prostate is not palpable. A urine
dipstick shows blood.
You answered Bulbar urethral rupture
The correct answer is Membranous urethral rupture
A pelvic fracture and highly displaced prostate should indicate a
diagnosis of membranous urethral
rupture.
9. A 52 year old woman falls out of a tree while rescuing a cat. She has a
pelvic fracture. She has
suprapubic tenderness and complains of dysuria. Her abdomen is soft
and non tender. A urine dipstick
shows blood, nitrites and leucocytes.
You answered Bladder contusion
The correct answer is Urinary tract infection
There is no indication of a more sinister diagnosis here. The patient's
abdomen is normal and she is
able to pass urine. Her dipstick confirms an infection. Also in women
urethral injury is rare.
Similar theme questions in September 2009 and April
2010

A 22 year old man is involved in a road traffic accident. He is found to


have a pelvic fracture. While on the
ward the nursing staff report that he is complaining of lower abdominal
pain. On examination you find a
distended tender bladder. What is the diagnosis?
A. Bladder rupture
B. Ureter injury
C. Urethral injury
D. Clot retention
E. Prostate rupture
Theme from 2009 Exam
Pelvic fractures may cause laceration of the urethra. Urinary retention,
blood at the urethral meatus and a high
riding prostate on digital rectal examination are the typical features.

Theme: Testicular disorders


A. Testicular tumour
B. Torsion of the spermatic cord
C. Acute infective epididymo-orchitis
D. Non infective epididymo- orchitis
E. Torsion of testicular appendage
F. Hydrocele
G. Haematocele
Please select the most likely cause for the testicular disorder described.
Each option may be used once, more
than once or not at all.
29. An 28 year old man presents with pain in the testis and scrotum. It
began 10 hours previously and has
worsened during that time. On examination he is pyrexial, the testis is
swollen and tender and there is
an associated hydrocele.
Acute infective epididymo-orchitis
The onset is relatively slow for torsion and the presence of fever favors
epididymo-orchitis.
30. A 15 year old boy develops sudden onset of pain in the left
hemiscrotum. He has no other urinary
symptoms. On examination the superior pole of the testis is tender and
the cremasteric reflex is
particularly marked.
Torsion of testicular appendage
The cremasteric reflex is usually preserved when the torsion affects the
appendage only.
31. A 14 year old boy develops sudden onset severe pain in the left
testicle radiating to the left groin. He
is distressed and vomits. On examination the testis is very tender and
the cremasteric reflex is absent.
Torsion of the spermatic cord
The cremasteric reflex is usually absent in torsion.
Similar theme question in September 2011 exam

Theme: Renal imaging


A. Non contrast abdominal CT scan
B. DMSA scan
C. PET/CT scan
D. MAG 3 Renogram
E. Renal ultrasound scan
F. DTPA Scan
G. Micturating cystourethrogram
H. Intra venous urography
Please select the most appropriate imaging modality for the scenario
descrived. Each agent may be used once,
more than once or not at all.
48. A 43 year old female has undergone a renal transplant 12 months
previously. Over the past few weeks
there have been concerns about deteriorating renal function.
You answered DMSA scan
The correct answer is MAG 3 Renogram
Theme from 2010 Exam
Similar theme September 2012 Exam
Because it is excreted by renal tubular cells a MAG 3 renogram
provides excellent imaging of renal
function and is often used in investigating failing transplants.
49. A 5 year old boy presents with recurrent urinary tract infections
and left sided loin pain. On investigation
he is found to have a left sided PUJ obstruction, there are concerns that
he may have developed renal
scarring.
You answered Non contrast abdominal CT scan
The correct answer is DMSA scan
Although MAG 3 renograms may provide some information relating to
the structural integrity of the
kidney, many still consider a DMSA scan to be the gold standard for the
detection of renal scarring
(which is the main concern in PUJ obstruction and infections).
50. A 17 year old man is referred to the urology clinic. As a child he was
diagnosed as having a right sided
PUJ obstruction. However, he was lost to follow up. Over the past 7
months he has been complaining of
recurrent episodes of right loin pain. A CT scan shows considerable
renal scarring.
You answered DTPA Scan
The correct answer is MAG 3 Renogram
In patients with long standing PUJ obstruction and renal scarring the
main diagnostic question is whether
the individual has sufficient renal function to consider a pyeloplasty or
whether a primary nephrectomy is
preferable. Since the CT has demonstrated scarring there is no use in
obtaining a DMSA scan. Of the
investigations listed both a DTPA and MAG 3 renogram will allow
assessment of renal function.
However, MAG 3 is superior in the assessment of renal function in
damaged kidneys (as it is subjected to
tubular secretion).

Theme: Vascular disorders affecting the upper limb


A. Proximal brachial artery occlusion secondary to atheroma
B. Distal brachial artery occlusion secondary to atheroma
C. Axillary artery embolus
D. Axillary vein thrombosis
E. Cervical rib
F. Raynaud's disease
G. Rheumatoid disease
Please select the most likely cause for the presenting scenario described.
Each option may be used once, more
than once or not at all.
7. A 73 year old male presents with a collapse and is brought to the
emergency department. On
examination he has a cold, painful left hand and forearm.
You answered Proximal brachial artery occlusion secondary to
atheroma
The correct answer is Axillary artery embolus
Theme from September 2012 Exam
Sudden arterial embolus will affect the axillary artery in up to 30%
cases. Because of the acute nature
of the condition there is no time for the development of a collateral
circulation so the limb is usually
pale and painful. Emboli occur usually occur as a result of atrial
fibrillation. Fast atrial fibrillation can
cause syncope and an acute embolus.
8. A 23 year old man presents with intermittent symptoms of altered
sensation in his arm and discomfort
when he uses his hands. He works as an electrician and his symptoms
are worst when he is fitting light
fixtures.
Cervical rib
Compression of the thoracic outlet by the fibrous band of the "rib" can
result in both neurological and
circulatory compromise. When manual tasks are performed in which
the hand works overhead the
signs and symptoms will be maximal and this is the basis of Adsons test.
9. A 19 year old lady presents with recurrent episodes of pain in her
hands. She notices that her
symptoms are worst in cold weather. When she gets the pain she notices
that her hands are very pale,
they then become dark blue in colour.
Raynaud's disease
Raynauds disease is characterised by a series of colour changes and
discomfort is often present. The
young age at presentation coupled with the absence of a smoking history
(in most cases) makes
occlusive disease unlikely.
Theme: Management of occlusive vascular disease
A. Aorto-bifemoral bypass graft
B. Femoro-femoral cross over graft
C. Femoro-popliteal bypass graft
D. Femoro-distal bypass graft
E. Axillo-bifemoral bypass graft
F. Bilateral above knee amputation
Please select the most appropriate arterial bypass method for the
scenario described. Each option may be used
once, more than once or not at all.
10. An 83 year old lady with a significant cardiac history is admitted
with rest pain and bilateral leg
ulcers. Imaging demonstrates bilateral occlusion of both common iliac
arteries that are unsuitable for
stenting.
Axillo-bifemoral bypass graft
Theme from January 2012 Exam
In patients with major cardiac co-morbidities the safest option is to
choose an axillo-bifemoral bypass
graft. The long term patency rates are less good than with aorto-
bifemoral bypass grafts, however, the
operation is less major.
11. A 54 year old man presents to the vascular clinic with severe rest
pain and an ulcer on his right foot
that is not healing. On examination he has bilateral absent femoral
pulses. Imaging demonstrates a
bilateral occlusion of the common iliac arteries that is not suitable for
stenting.
Aorto-bifemoral bypass graft
In a young patient consideration should be given to aorto-bifemoral
bypass grafts as these have the
best long term functional outcome compared with an axillobifemoral
bypass graft.
12. A 78 year old man presents with left sided rest pain in his leg and a
non healing arterial leg ulcer on
the same leg. Imaging shows normal right leg vessels, on the left side
there is a long occlusion of the
external iliac artery that is unsuitable for stenting. He has a significant
cardiac history.
Femoro-femoral cross over graft
Femoro-femoral cross over grafts are an option for treatment of iliac
occlusions in patients with
significant co-morbidities and healthy contralateral vessels. In reality
the idealised situation presented
here seldom applies and the opposite vessels usually have some disease
and one must be careful not
to damage the "healthy" side.

Theme: Ankle brachial pressure index


A. >1.2
B. 1.0
C. 0.8
D. 0.5
E. 0.3
Please select the ankle brachial pressure index that is most likely to be
present for the scenario given. Each
value may be used once, more than once or not at all.
13. A 73 year old lifelong heavy smoker presents to the vascular clinic
with symptoms of foot ulceration
and rest pain. On examination her foot has areas of gangrene and pulses
are impalpable.
0.3
This is critical limb ischaemia. Values of 0.3 are typical in this setting
and urgent further imaging is
needed. Debridement of necrosis prior to improving arterial inflow
carries a high risk of limb loss.
14. A 63 year old man presents with a claudication distance of 15 yards.
He is a lifelong heavy smoker.
On examination his foot is hyperaemic and there is a small ulcer at the
tip of his great toe.
0.5
Hyperaemia may occur in association with severe vascular disease and
is referred to surgically as a
"sunset foot". ABPI is usually higher than 0.3, but seldom greater than
0.5. Especially when
associated with hyperaemic changes and ulceration. Urgent further
imaging and risk factor
modification is needed.
15. A 77 year old morbidly obese man with type 2 diabetes presents with
leg pain at rest. His symptoms
are worst at night and sometimes improve during the day. He has no
areas of ulceration.
You answered 0.8
The correct answer is >1.2
Type 2 diabetes may have vessel calcification. This will result in
abnormally high ABPI readings.
Pain of this nature in diabetics is usually neuropathic and if a duplex
scan is normal then treatment
with an agent such as duloxetine is sometimes helpful.
Theme from September 2011 and September 2012 exam

Theme: Lower limb ulceration


A. Mixed ulcer
B. Chronic obliterative arterial disease
C. Superficial venous insufficiency
D. Deep venous insufficiency
E. Neuropathic ulcer
F. Basal cell carcinoma
G. Squamous cell carcinoma
Please select the most likely cause of ulceration for the scenario given.
Each option may be used once, more
than once or not at all.
34. A 65 year old diabetic female presents with a painless ulcer at the
medial malleolus, it has been
present for the past 16 years. On examination she has evidence of
truncal varicosities and a brownish
discolouration of the skin overlying the affected area.
Superficial venous insufficiency
Theme from September 2012 Exam
Venous ulcers are usually associated with features of venous
insufficiency. These include
haemosiderin deposition and varicose veins. Neuropathic ulcers will
tend to present at sites of
pressure, which is not typically at the medial malleolus.
35. A 71 year old man presents with a painful lower calf ulcer, mild
pitting oedema and an ABPI of 0.3.
Chronic obliterative arterial disease
Painful ulcers associated with a low ABPI are usually arterial in nature.
The question does not
indicate that features of chronic venous insufficiency are present.
Patients may have mild pitting
oedema as many vascular patients will also have ischaemic heart disease
and elevated right heart
pressures. The absence of more compelling signs of venous insufficiency
makes a mixed ulcer less
likely.
36. A 79 year old retired teacher has had an ulcer for 15 years. It is at
the medial malleolus and has
associated lipodermatosclerosis of the lower limb. The ulcer base is
heaped up and irregular.
Squamous cell carcinoma
If after many years an ulcer becomes heaped up and irregular, with
rolled edges then suspect a squamous cell carcinoma.

A 33 year old lady attends the clinic with a 3 month history of


palpitations and irritability. Her thyroid function,
PTH and calcium are measured:
Thyroid function
Free T4 40 pmol/L
TSH < 0.1 miu/L
Free T3 25 p mol/L (normal 3.5-7.7 p mol/L)
PTH 10pg/ml (normal 10-55pg/ml)
(Normal values listed in reference range link)
What is the most likely diagnosis?
A. Hypothyroidism
B. Hyperthyroidism
C. Hypoparathyroidism
D. Hyperparathyroidism
E. Euthyroid
Theme from April 2012 Exam
Elevated T4 and suppressed TSH makes this the most likely diagnosis.
The PTH level is normal.

Theme: Thyroid function tests


A. Thyrotoxicosis
B. Sick euthyroid syndrome
C. Hypothyroidism
D. Poor compliance with thyroid medication
E. Hashimotos thyroiditis
F. Multinodular goitre
Please select the most likely diagnosis for the scenario given. Each
option may be used once, more than once or
not at all.
31. A 52 year old woman presents with an acute ischaemic right arm.
She is found to have fast atrial
fibrillation. Her blood results reveal a free T4 level of 20 and a TSH of <
0.01.
Thyrotoxicosis
The diagnosis is thyrotoxicosis. An elevated T4 and a low TSH should
indicate this diagnosis.
Remember atrial fibrillation and its complications i.e acute ischaemic
limbs can be precipitated by
hyperthyroid disorders.
32. A 42 year old woman presents with a goitre. On examination the
goitre feels 'lumpy'. The blood
results reveal a TSH of 12 and a free T4 of 2. Antithyroid peroxidase
antibodies are high.
Hashimotos thyroiditis
Hashimotos usually presents in women aged between 30- 50 years. They
are normally associated with
a goitre. To differentiate from hypothyroidism, the antithyroid
peroxidase antibodies will be elevated.
33. A 55 year old man is on the intensive care unit for many months
after open aortic surgery. He is
maintained on total parenteral nutrition. Clinically he is euthyroid, but
his thyroid function tests
reveal a low TSH and low T4.
Sick euthyroid syndrome
Sick euthyroid syndrome is most commonly seen in chronically ill
patients or those with starvation.
The thyroid function tests are often low and the patient clinically
euthyroid.

Theme: Management of ano-rectal disease


A. Excision and primary closure
B. Incision and drainage
C. Topical steroids
D. Topical diltiazem
E. Steroid injections
F. Haemorroidectomy
G. Manual anal dilation
H. Injection with 88% aqueous phenol
I. Sphincterotomy
Please select the most appropriate management for the situation
described. Each option may be used
once, more than once or not at all.
1. A 19 year old female presents with severe anal pain and bleeding
which typically occurs post
defecation. On examination she has a large posteriorly sited fissure in
ano.
You answered Manual anal dilation
The correct answer is Topical diltiazem
Theme from January 2013 Exam
Initial therapy should be with pharmacological agents to relax the
sphincter and facilitate
healing. This is particularly true in females presenting for the first time.
2. A 43 year old male has been troubled with symptoms of post
defecation bleeding for many
years. On examination he has large prolapsed haemorroids, colonoscopy
shows no other
disease.
Haemorroidectomy
Prolapsed haemorroids are best managed surgically if symptomatic.
Note that phenol injections
are usually only used for minor internal haemorroids. Where used low
concentration phenol in
oil is used, the phenolic solution above is used to ablate the nail bed in
toe nail surgery!
3. A 20 year old man presents with a 24 hour history of anal pain. On
examination he has a peri
anal abscess.
Incision and drainage
Abscesses require incision and drainage as a first line treatment.

A 70 year old female is admitted with a history of passing brown


coloured urine and abdominal
distension. On examination she is clinically got large bowel obstruction
with central abdominal
tenderness. She is maximally tender in the left iliac fossa. The is no
evidence of haemodynamic
instability. What is the most appropriate investigation?
A. Cystogram
B. Abdominal X-ray of the kidney, ureters and bladder
C. Computerised tomogram of the abdomen and pelvis
D. Flexible sigmoidoscopy
E. Barium enema
Theme from April 2013 Exam
This lady is most likely to have a colovesical fistula complicating
diverticular disease of the sigmoid
colon. In addition she may also have developed a diverticular stricture
resulting in large bowel
obstruction. A locally advanced tumour of the sigmoid colon may
produce a similar clinical picture. The
best investigation of this acute surgical patient is an abdominal CT scan,
this will demonstrate the site of
the disease and also supply regional information such as organ
involvement and other local
complications such as a pericolic abscess. A barium enema would
require formal bowel preparation and
this is contra indicated where large bowel obstruction is suspected. A
flexible sigmoidoscopy is unlikely
to be helpful and the air insufflated at the time of endoscopy may make
the colonic distension worse. A
cystogram would provide only very limited information.

A 53 year old man presents with a full thickness external rectal prolapse.
Which of the following
procedures would be the most suitable surgical option?
A. Rectopexy
B. Delormes
C. Altmeirs
D. Thirsch tape
E. Abdomino-perineal excision of the rectum
Theme from 2012 Exam
As this man is young and has full thickness prolapse a rectopexy is the
most appropriate procedure. It
will give the lowest recurrence rates.

You might also like